2
$\begingroup$

Possible Duplicate:
Calulus of variations Euler Lagrange equation

Find the extremals for the following functionals and discuss whether they provide a minimum or a maximum to the functional.

$j[y]=\int_0^1 \exp(y)(y')^2 dx $ subject to the boundary conditions $y(0)=1$ and $y(1)=log(4)$.

I am aware that you have to find the euler lagrange equation to the problem but I am then having trouble solving it for y' and y. Also how do I know whether it provides a min or a max? I'm pretty sure I need to use this, $ j[y+f] = \int_0^1(y'+f')^2-(y+f) dx $ Any help would be greatly appreciated.

  • 0
    This seems to be exactly the same question as http://math.stackexchange.com/questions/218121/calulus-of-variations-euler-lagrange-equation2012-10-21

0 Answers 0